Laplace_Transform
Laplace_Transform
Jeeja A. V.
Assistant Professor
Department of Mathematics
Govt. K.N.M Arts and Science College, Kanjiramkulam
E-Resource of Mathematics
Contents
1 Laplace transform 3
6 Periodic Functions 32
7 Convolution 34
References 56
Jeeja A. V. [email protected]
3
1 LAPLACE TRANSFORM
Rb
A relation of the form F(s) = a k(s,t) f (t) dt which transforms a given function f (t) into
another function F(s), is called an integral transform. Here K(s,t) is called the kernal of the
transform and F(s) the transform of f (t). The most common integral transforms are
The idea behind any transform is that given problem can be solved more easily in the trans-
formed domain. Laplace transform reduces the problem of solving a differential equation to an
algebraic problem. It is widely used in problems where the
Definition 1.1
Let f (t) be a function defined for all t ≥ 0. The Laplace transform of f (t) is defined as
Z ∞
L [ f (t)] = e−st f (t) dt = F(s), (1)
0
provided the integral exists. Here s is a parameter real or complex. The function f (t)
whose transform is F(s) is said to be the inverse transform of F(s) and is denoted by
L −1 [F(s)]. Thus if L [ f (t)] = F(s), then f (t) = L −1 [F(s)].
A function f (t) is said to be of exponential order or satisfies growth restriction if there exist
constants M and a such that | f (t)| ≤ M eat for all positive t.
Proof. Since f (t) is piecewise continuous, e−st f (t) is integrable over any finite interval on the
Jeeja A. V. [email protected]
4
I Note that Z ∞ Z k
f (x) dx = lim f (x) dx
a k→∞ a
The Laplace transform is a linear operation. i.e. for any function f (t) and g(t) whose Laplace
transform exist and any constants a and b,
Proof. By definition,
Z ∞
L [a f (t) + bg(t)] = e−st [a f (t) + bg(t)] dt
0Z
∞ Z ∞
−st
=a e f (t) dt + b e−st g(t) dt
0 0
= aL [ f (t)] + bL [g(t)].
Jeeja A. V. [email protected]
5
1
(i) L (1) = , s > 0.
s
Proof. By definition,
Z ∞
L (1) = e−st 1 dt
0
Z k
= lim e−st 1 dt
k→∞ 0
−st k
e
= lim
k→∞ −s 0
−sk
e 1
= lim +
k→∞ −s s
1
= 0+
s
1
= , s > 0.
s
1
(ii) L (eat ) = , s > a.
s−a
Proof.
Z ∞
L [e ] =
at
e−st eat dt
Z0 ∞
= e−(s−a)t dt
0
" #k
e−(s−a)t
= lim
k→∞ −(s − a)
0
" #
e−(s−a)k 1
= lim +
k→∞ −(s − a) s−a
1
= if s > a.
s−a
n!
(iii) L (t n ) = where n is a positive integer.
sn+1
1
Proof. We prove this formula by induction. For n = 0, t n = t 0 = 1 and L (1) = . Thus
s
the result is true for n = 0. We now make the induction hypothesis that it holds for any
n!
positive integer n. i.e. L (t n ) = n+1 .
s
Jeeja A. V. [email protected]
6
Now
Z ∞
L (t n+1
)= e−st t n+1 dt
Z0∞
= t n+1 e−st dt
0
Z k
= lim t n+1 e−st dt
k→∞ 0
" #
e −st k Z k
e−st
= lim t n+1 − (n + 1)t n dt
k→∞ −s 0 0 −s
−sk Z k −st
n+1 e ne
= lim k − 0 − (n + 1)t dt
k→∞ −s 0 −s
n + 1 ∞ −st n
Z
= [0 − 0] + e t dt
s 0
n+1
= L (t n )
s
n + 1 n!
=
s sn+1
(n + 1)!
= n+2 .
s
− cos at − cos at
Z
I = e−st − (−s)e−st dt
a a
−e−st cos at s
Z
= − e−st cos at dt
a a
−e−st cos at
s −st sin at −st sin at
Z
= − e − (−s)e dt
a a a a
−e−st cos at s −st s2
Z
= − 2 e sin at − 2 e−st sin at dt
a a a
−e−st cos at s −st s2
= − 2 e sin at − 2 I
a a a
s2 −e−st cos at s −st
⇒I+ I= − 2 e sin at
a2 a a
(s2 + a2 )I −st
−e cos at s
⇒ = − 2 e−st sin at
a2 a a
⇒ (s2 + a2 )I = −ae−st cos at − se−st sin at
Jeeja A. V. [email protected]
7
Hence,
Z k
L [sin at] = lim e−st sin at dt
k→∞ 0
k
e−st
= lim 2 (−s sin at − a cos at)
k→∞ s + a2 0
−sk
e 1
= lim 2 (−s sin ak − a cos ak) − 2 (0 − a)
k→∞ s + a2 s + a2
a
= 0+ 2
s + a2
a
= 2 , s > 0.
s + a2
Proof. We have
sin ax 1
e−sx sin ax = sx
≤ sx = e−sx .
e e
s
(v) L (cos at) = , s > 0.
s2 + a2
Z k
Proof. L [cos at] = lim e−st cos at dt. As in the previous problem, we can see that
k→∞ 0
R −st e−st
e cos at dt = s2 +a2
(−s cos at + a sin at). Using it,
Z ∞
L [cos at] = e−st cos at dt
0
−st k
e
= lim 2 (−s cos at + a sin at)
k→∞ s + a2 0
−sk
e 1
= lim 2 (−s cos ak + a sin ak) − 2 (−s + 0)
k→∞ s + a2 s + a2
s
= 0+ 2
s + a2
s
= 2 .
s + a2
a
(vi) L [sinh at] = , s > |a|
s2 − a2
Jeeja A. V. [email protected]
8
eat − e−at
Proof. We have sinh at =
2
1 +at −at
L [sinh at] = L (e − e )
2
1
= {L (eat ) − L (e−at )}
2
1 1 1
= −
2 s − a s − (−a)
1 s + a − (s − a)
=
2 (s − a)(s + a)
1 2a
= × 2
2 s − a2
a
= 2 .
s − a2
s
(vii) L [cosh at] = , s > |a|
s2 − a2
Proof.
eat + e−at
L [cosh at] = L
2
1
= {L (eat ) + L (e−at )}
2
1 1 1
= +
2 s−a s+a
1 s+a+s−a
=
2 s2 − a2
1 2s
= × 2
2 s − a2
s
= 2 .
s − a2
Problem 2.1
et , 0 < t < 1
Find the Laplace transform of f (t) =
0 when t > 1
Jeeja A. V. [email protected]
9
Solution. We have
Z ∞
L { f (t)} = e−st f (t) dt
0
Z 1 Z ∞
−st
= e f (t) dt + e−st f (t) dt
0 1
Z 1 Z ∞
= e−st et dt + e−st 0 dt
0 1
Z 1
= e−(s−1)t dt
0
−(s−1)t 1
e
=
−(s − 1) 0
e−(s−1) 1
= −
−(s − 1) −(s − 1)
e1−s 1
= −
1−s 1−s
e1−s − 1
= .
1−s
Problem 2.2
cost, 0 < t < 2π
Find the Laplace transform of f (t) =
0, t > 2π
Solution. We have
Z ∞
L { f (t)} = e−st f (t) dt
0
Z 2π Z ∞
−st
= e f (t) dt + e−st f (t) dt
0 2π
Z 2π Z ∞
= e−st cost dt + e−st 0. dt
0 2π
2π
e−st
= (−s cost + sint)
s2 + 1 0
e−s2π 1
= 2 (−s) − 2 (−s)
s +1 s +1
s
= (1 − e−2sπ ) 2 .
s +1
Problem 2.3
Find the Laplace transform of the following functions.
(i) sin 3t cos 2t (iii) cos3 2t (v) sin 3t sin 2t
(ii) sin2 3t (iv) sinh3 2t (vi) e−4t − 6t 2 + 4 sin 2t
Jeeja A. V. [email protected]
10
Solution. 1. We have
1
L [sin 3t cos 2t] = {L (sin 5t) + L (sint)}
2
1 5 1
= +
2 s2 + 25 s2 + 1
3(s2 + 5)
= 2
(s + 1)(s2 + 2s)
2. we have
1 − cos 6t 1
L (sin 3t) = L
2
= {L (1) − L (cos 6t)}
2 2
1 1 s 18
= − 2 = 2
2 s s + 36 s(s + 36)
1 3
3. we have cos 3A = 4 cos3 A − 3 cos A or cos3 A = cos 3A + cos A
4 4
1 3
∴ cos3 2t = cos 6t + cos 2t
4 4
1 3
L (cos3 2t) = L (cos 6t) + L (cos 2t)
4 4
1 s 3 s
= × 2 + × 2
4 s + 36 4 s + 4
s(s2 + 28)
=
(s2 + 4)(s2 + 36)
1
5. We have sin A sin B = [cos(A − B) − cos(A + B)]
2
1
i.e. sin 3t sin 2t = [cost − cos 5t]
2
1
∴ L [sin 3t sin 2t] = [L (cost) − L [(cos 5t)]
2
1 s s
= −
2 s2 + 1 s2 + 25
12s
= 2
(s + 1)(s2 + 25)
Jeeja A. V. [email protected]
11
6. We have
Problem 2.4
Find the Laplace transforms of
2. sin(at + b)
Solution. 1. L [3e5t + (t + 2)2 + 2 cos 3t] = 3L (e5t ) + L [(t + 2)2 ] + 2L [cos 3t] (1)
1
But L [e5t ] =
s−5
L ((t + 2)2 ) = L [t 2 + 4t + 4] = L (t 2 ) + 4L (t) + L (4)
2! 1! 1 2 4 4
= 3
+4· 2 +4× = 3 + 2 + .
s s s s s s
s
and L [cos 3t) = 2
s +9
Substituting in (1)
3 2 4 4 2s
L [3e5t + (t + 2)2 + 2 cos 3t] = + 3+ 2+ + 2
s−5 s s s s +9
EXERCISE
Jeeja A. V. [email protected]
12
t/k, 0 < t < k
(1) f (t) =
1, t > k
sint, 0 < t < π
(2) f (t) =
0, t > π
(6) sin3 2t
(7) cos(at + b)
(8) cosh3 2t
ANSWERS
1 − e−ks 1 + e−πs s(s2 + 5)
(1) (2) (3)
ks2 s2 + 1 (s2 + 1)(s2 + 9)
2(s2 − 5) 5 18 3(s − 2)
(4) (5) + 4+ 2
(s2 + 1)(s2 + 25) s−3 s s +9
48 s cos b − a sin b
(6) (7)
(s2 + 4)(s2 + 36) s2 + a2
s(s2 − 28) 2
s + 18 24 4 1
(8) (9) + 5 + 3+
(s2 − 4)(s2 − 36) 2
s(s + 36) s s s
Jeeja A. V. [email protected]
13
Proof. We have
Z ∞
F(s) = L [ f (t)] = e−st f (t) dt (1)
Z0 ∞
∴ L [eat f (t)] = e−st eat f (t) dt
Z0∞
= e−(s−a)t f (t) dt
0
(i) e−3t t 3
(v) et cosh 3t
(vii) (t + 1)2 et
3! 6
Solution. (i) We have L [t 3 ] = 4
= 4 . By shifting property, we get
s s
6
L [e−3t t 3 ] = (Replacing s by s + 3)
(s + 3)4
1 + cos 2t
(ii) We have cos2 t =
2
1 1 1 s
∴ L [cos t] = L [1 + cos 2t] =
2
+ .
2 2 s s2 + 4
By shifting property,
−2t 1 1 s+2
L [e 2
cos t] = + (Replacing s by s + 2)
2 s + 2 (s + 2)2 + 4
Jeeja A. V. [email protected]
14
eat − e−at
(iii) We have sinh at = so that
2
eat − e−at
sinh at · sin at = sin at
2
1
= [eat sin at − e−at sin at]
2
1
∴ L [sinh at sin at] = L [eat sin at] − L [e−at sin at]} (1)
2
a a
But L [sin at] = 2 2
. By shifting property, L [eat sin at] = and L [e−at sin at] =
s +a (s − a)2 + a2
a
.
(s + a)2 + a2
Substituting in (1),
1 a a
L [sinh at sin at] = −
2 (s − a)2 + a2 (s + a)2 + a2
s 4
(iv) We have L [cos 4t + 3 sin 4t] = +3×
s2 + 16 s2 + 16
12 + s
= .
s2 + 16
By shifting property,
12 + (s + 2) s + 14
L [e−2t (cos 4t + 3 sin 4t)] = =
(s + 2)2 + 16 s2 + 4s + 20
e2t − e−2t
5
(vi) 5e2t sinh 2t = 5e2t = [e4t − 1]
2 2
5 5 1 1
∴ L [5e sinh 2t] = [L (e ) − L (1)] =
2t 4t
−
2 2 s−4 s
10
=
s(s − 4)
or
2 10
We have L [5 sinh 2t] = 5 × = .
s2 − 4 s2 − 4
Jeeja A. V. [email protected]
15
By shifting property,
10 10
L [e2t 5 sinh 2t] = =
(s − 2)2 − 4 s2 − 4s + 4 − 4
10
=
s(s − 4)
2! 1 1 2 + 2s + s2
= + 2 × + =
s2 s2 s s3
2 + 2(s − 1) + (s − 1)2
By shifting property, L [et (t + 1)2 ] =
(s − 1)3
s2 + 1
= .
(s − 1)3
Problem 2.6
1 s
If L [ f (t)] = F(s), then show that L [ f (at)] = F
a a
Proof.
Z ∞
L [ f (at)] = e−st f (at) dt put at = u a dt = du
0
du
Z ∞
= e−su/a · f (u) ·
0 a
1
= F(s/a).
a
Jeeja A. V. [email protected]
16
Theorem 3.1
If L [ f (t)] = F(s), then the transforms of the first and second derivatives of f (t) satisfy
The first equation holds if f (t) is continuous for all t ≥ 0 and satisfies the growth re-
striction and f 0 (t) is piecewise continuous on every finite interval on the semi-axis t ≥ 0.
Similarly, second equation holds if f and f 0 are continuous for all t ≥ 0 and satisfy the
growth restriction and f 00 is piecewise continuous on every finite interval on the semi-axis
t ≥ 0.
Proof. By definition,
Z ∞
L [ f 0 (t)] = e−st f 0 (t)dt
0
Integrating by parts
0
−st k Z ∞ −st
L [ f (t)] = lim e f (t) 0 − e (−s) f (t)dt
k→∞ 0
h i Z∞
−sk
= lim e f (k) − f (0) − e−st (−s) f (t)dt
k→∞ 0
Z ∞
= 0 − f (0) + s e−st f (t)dt
0
= − f (0) + s · L [ f (t)].
Thus
L [ f 0 (t)] = sL [ f (t)] − f (0).
Hence
L [ f 00 (t)] = s2 F(s) − s f (0) − f 0 (0).
Jeeja A. V. [email protected]
17
Theorem 3.2
Let f , f 0 , . . . , f (n−1) be continuous for all t ≥ 0 and satisfy the growth restriction. Fur-
thermore, let f be piecewise continuous on every finite interval on the semi-axis t ≥ 0.
Then the transform of f (n) satisfies
L [ f (n) ] = sn L [ f ] − sn−1 f (0) − sn−2 f 0 (0) − · · · − s f (n−2) (0) − f (n−1) (0). (2)
TRANSFORMS OF INTEGRALS
Theorem 3.3
Let F(s) denote the transform of a function f (t) which is piecewise continuous for t ≥ 0
and satisfies a growth restriction. Then, f ors > 0, s > k, and t > 0,
Z t Z t
F(s) −1 F(s)
L f (u)du = , f (u)du = L (3)
0 s 0 s
Proof. By definition,
Z t Z ∞
Z t
−st
L f (u)du = e f (u)du dt
0 0 0
Z ∞ Z t
= f (u)du e−st dt
0 0
Z k Z t
= lim f (u)du e−st dt
k→∞ 0 0
" Z −st k Z k #
t e e−st
= lim f (u)du − f (t) dt
k→∞ 0 −s 0 0 −s
−sk Z 0
e−st
Z k Z k
e 1
= lim f (u)du − f (u)du − f (t) dt
k→∞ 0 −s 0 −s 0 −s
0 1 e−st
Z ∞
= L { f (t)} − (0) − f (t) dt
−s −s 0 −s
1 ∞ −st
Z
= (0 − 0) + e f (t)dt
s 0
F(s)
= .
s
Problem 3.1
1
If L [ f (t)] = , find f (t)
s(s2 + w2 )
Jeeja A. V. [email protected]
18
Solution. Let
1
F(s) =
s2 + w2
Then
F(s) 1
= 2
s s(s + w2 )
−1 1
∴ f (t) = L
s(s2 + w2 )
Zt
−1 F(s)
=L = L −1 [F(s)]dt
s 0
Z t
−1 1
= L dt
0 s2 + w2
1 − cos wt t
Z t
sin wt
= dt =
0 w w w 0
1 1 − cos wt
= − 2 [cos wt − 1] = .
w w2
Problem 3.2
Find
−1 1
L
s2 (s2 + w2 )
Solution. Let
1
F(s) =
s2 + w2
Then
−1 −1 1 sin wt
L (F(s)) = L =
s + w2
2 w
Now
Zt
−1 1 F(s)−1
L = L = L −1 [F(s)]ds
s(s2 + w2 ) s 0
1 − cos wt t
Z t
sin wt
= dt =
0 w w w 0
−1 1 − cos wt
= (cos wt − 1) =
w w2
1 G(s) 1
Let G(s) = , so = 2 2 .
s(s2 + w2 ) s s (s + w2 )
Zt
−1 1 −1 G(s)
∴L = L = L −1 [G(s)]dt
s2 (s2 + w2 ) s 0
sin wt t
Z t
1 − cos wt 1
= dt = 2 t −
0 w2 w w 0
1 sin wt
= 2
t− .
w w
Jeeja A. V. [email protected]
19
EXERCISE
Answers
1
(i) (1 − e−4t ) (ii) cosht − 1
4
1
(iii) 1 + t − cos 3t − sin 3t (iv) 1 − cost
3
1 1 − e−at
(v) [1 + 2 cos 3t] (vi)
3 a
(vii) cos2 t (viii) t − 1 + e−t
1
(ix) [cosh 4t − 1]
16
Let us now discuss how the Laplace transform method solves ODEs and initial value problems.
We consider an initial value problem
where a and b are constants. Here r(t) is the given input (driving force) applied to the mechani-
cal or electrical system and y(t) is the output (response to the input) to be obtained. In Laplace’s
method we do three steps:
where Y = L (y), R(s) = L (r). From this we get the subsidiary equation
Jeeja A. V. [email protected]
20
a2
2
a 2
s + as + b = s + + b−
2 4
and so we get
(s + a)y(0) + y0 (0) + R(s)
Y= 2 2
s + a2 + b − a4
Step 3: Reduce RHS of the previous equation (usually by partial fractions as in calculus) to a
sum of terms whose inverses can be found, so that we obtain the solution y(t) = L −1 (Y ).
Solution. Given equation is y00 + 2y0 + 5y = e−t sint. Taking Laplace transforms of both sides,
1
Also L (sint) =
s2 + 1
1
So L [e−t sint] = (by shifting theorem)
(s + 1)2 + 1
1
= 2
s + 2s + 2
1
[s2 L (y) − 1] + 2[sL (y)] + 5L (y) =
s2 + 2s + 2
1
i.e. (s2 + 2s + 5)L (y) = +1
s2 + 2s + 2
1 + s2 + 2s + 2
=
s2 + 2s + 2
s2 + 2s + 3
= 2
s + 2s + 2
(s2 + 2s + 3)
∴ L (y) =
(s2 + 2s + 2)(s2 + 2s + 5)
Jeeja A. V. [email protected]
21
1/3 2/3
L (y) = +
s2 + 2s + 2 s2 + 2s + 5
1/3 2/3
= +
(s + 1) − 1 + 2 (s + 1)2 − 1 + 5
2
1/3 2/3
= +
(s + 1) + 1 (s + 1)2 + 4
2
1 −1 1 2 −1 1
∴ y= L + L
3 (s + 1)2 + 1 3 (s + 1)2 + 22
1 −t 2 sin 2t
= e · sint + × e−t
3 3 2
1 −t
= [e sint + e−t sin 2t].
3
Problem 3.4
Using Laplace transforms, solve y00 + 3y0 + 2y = 8 cos 2t, given that y(0) = −1, y0 (0) = 2.
Solution. Given equation is y00 + 3y0 + 2y = 8 cos 2t. Taking Laplace transforms of both sides
8s
L (y)[s2 + 3s + 2] = −s−1
s2 + 4
8s − s3 − 4s − s2 − 4
i.e. L (y)(s2 + 3s + 2) =
s2 + 4
−s3 − s2 + 4s − 4
i.e. L (y) = 2 (1)
(s + 4)(s2 + 3s + 2)
Let
Jeeja A. V. [email protected]
22
Put s = −1,
1 − 1 − 4 − 4 = c(1 + 4)(−1 + 2)
−8 = 5c
⇒ c = −8/5
Put s = −2,
+8 − 4 − 8 − 4 = D(4 + 4)(−2 + 1)
−8 = −8D
⇒D = 1
−4 = 2B + 8C + 4D
64 24
⇒ 2B = −4 − 8C − 4D = −4 + −4 =
5 5
12
⇒B=
5
−2 12
s+
∴ L (y) = 5 2 5 + −8/5 + 1
s +4 s+1 s+2
−2 s 12 1 8 1 1
= 2
+ 2
− +
5 s + 4 5 s + 4 5 s + 1 s + 2
2 −1 s 12 −1 1 8 −1 1 −1 1
∴y=− L + L − L +L
5 s2 + 4 5 s2 + 4 5 s+1 s+2
2 12 sin 2t 8 −t
= − cos 2t + − e + e−2t .
5 5 2 5
Problem 3.5
d2 y dy
Using Laplace transform method solve 2
+ 2 + 6y = 6te−t , given that y(0) = 2,
dt dt
y0 (0) = 5.
Solution. Given equation is y00 + 2y0 + 6y = 6te−t . Taking Laplace transform of both sides,
Jeeja A. V. [email protected]
23
6
(s2 + 2s + 6)L (y) = + 2s + 9
(s + 1)2
2s3 + 13s2 + 20s + 15
=
(s + 1)2
2s3 + 13s2 + 20s + 15
∴ L (y) = (1)
(s + 1)2 (s2 + 2s + 6)
Put s = −1,
−2 + 13 − 20 + 15 = B(1 − 2 + 6)
6 = 5B ⇒ B = 6/5
A = 0, C = 2, D = 39/5
2s3 + 13s2 + 20s + 15 6/5 2s + 39/5
Therefore = +
(s + 1)2 (s2 + 2s + 6) (s + 1)2 s2 + 2s + 6
6/5 2(s + 1) − 2 + 39/5
= 2
+
(s + 1) (s + 1)2 + 5
6/5 (s + 1) 29/5
= 2
+2 +
(s + 1) (s + 1) + 5 (s + 1)2 + 5
2
Jeeja A. V. [email protected]
24
Substituting in (1),
6 1 s+1 29 1
L (y) = · +2 2
+
5 (s + 1)2 (s + 1) + 5 5 (s + 1)2 + 5
6 −1 1 −1 s+1 29 −1 1
or y = L + 2L + L
5 (s + 1)2 (s + 1)2 + 5 5 (s + 1)2 + 5
√
6 √ 29 sin 5t
= te−t + 2e−t cos 5t + × e−t √
5 5 5
−t √ √
e 29
= 6t + 10 cos 5t + √ sin 5t .
5 5
Problem 3.6
d2 y dy
Solve the differntial equation by using Laplace transform − 3 + 2y = 4, given
dt 2 dt
y(0) = 2, y0 (0) = 3.
Solution. Given equation is y00 − 3y0 + 2y = 4. Taking Laplace transforms of both sides,
4 2s2 − 3s + 4
L (y)[s2 − 3s + 2] = + 2s − 3 =
s s
2
2s − 3s + 4 2s2 − 3s + 4
L (y) = 2 = (1)
s(s − 3s + 2) s(s − 1)(s − 2)
2
2s − 3s + 4 A B C
Let = + +
s(s − 1)(s − 2) s s − 1 s − 2
2.02 − 3.0 + 4 4
where A = = =2
(0 − 1)(0 − 2) 2
2.12 − 3.1 + 4
B= = −3
1(1 − 2)
2.22 − 3 × 2 + 4 8 − 6 + 4
C= = =3
2(2 − 1) 2×1
2s2 − 3s + 4 2 −3 3
∴ = + +
s(s − 1)(s − 2) s s − 1 s − 2
Substituting in (1)
2 3 3
L (y) = − +
s s −1 s − 2
−1 1 −1 1 −1 1
∴ y = 2L − 3L + 3L
s s−1 s−2
= 2 − 3et + 3e2t .
Jeeja A. V. [email protected]
25
EXERCISE
ANSWERS
1
(1) y = 3 + 2t + (e3t − et ) − 2e2t
2
1
(2) x = 2et − 3tet + t 2 et
2
1
(3) y = (7e−t − 3e−3t − 2te−t )
4
1 1 1
(4) y = et − e−3t − (2 sint + cost)
8 40 10
5 1
(5) y = et − e−t + e−2t
3 3
(6) y = e−t (1 − t + t 3 /6)
Jeeja A. V. [email protected]
26
e−t 2
(8) y = (t + 2t)
2
(9) y = 2 + 2et − e−t − sint
1 4 4
(10) y = cos 2t + cos 3t + sin 3t
5 5 5
20 −t/2 11 −2t 1 −2t
(11) y = e − e − te
9 9 3
(12) y = 2e−t cost + e−t sint − 2 cost + sint
Jeeja A. V. [email protected]
27
Problem 4.1
Express the following function in terms of unit step function and find its Laplace trans-
form
8, t < 3
f (t) =
5, t > 3
Solution.
8 + 0, t < 3
f (t) =
8 − 3, t > 3
0, t < 3
= 8+
−3, t > 3
0, t < 3
= 8 + (−3)
1, t > 3
= 8 − 3u(t − 3)
Proof.
= e−st f (t − a)dt
0
Jeeja A. V. [email protected]
28
= e−as F(s).
and
0 if t < a,
f (t − a)u(t − a) =
f (t − a) if t > a.
Problem 4.2
Express the following function in terms of unit step function and hence find its Laplace
transform
2 if 0<t <π
f (t) = 0 if π < t < 2π
sint if t > 2π.
Jeeja A. V. [email protected]
29
f (t) = 2[u(t − 0) − u(t − π)] + 0[u(t − π) − u(t − 2π)] + sint · u(t − 2π)
Problem 4.3
Express the following function in terms of unit step function
2 + t 2 if 0 < t < 2
f (t) = 6 if 2 < t < 3
2
if t > 3.
2t − 5
2
f (t) = (2 + t 2 )[u(t − 0) − u(t − 2)] + 6[u(t − 2) − u(t − 3)] + · u(t − 3)
2t − 5
32 − 12t
= (2 + t 2 )u(t) + (4 − t 2 ) u(t − 2) + · u(t − 3).
2t − 5
Problem 4.4
2 2e−2s 4e−2s se−πs
Find the inverse transform of F(s) = − 2 − + 2 .
s2 s s s +1
1 1 s
Solution. We have L −1 = t, L −1 = 1 and L −1 = cost. Now
s2 s s2 + 1
Jeeja A. V. [email protected]
30
Thus
2 2e−2s 4e−2s se−πs
−1 −1
L [F(s)] = L − 2 − + 2
s2 s s s +1
−2s −2s −πs
−1 1 −1 e −1 e −1 se
= 2L − 2L − 4L +L
s2 s2 s s2 + 1
= 2t − 2(t − 2)u(t − 2) − 4u(t − 2) + cos(t − π)u(t − π)
= 2t − 2tu(t − 2) − costu(t − π)
2t if 0 < t < 2
= 0, if 2 < t < π
− cost if t > π
Problem 4.5
Find the inverse Laplace transform of the following:
e−πs
(i)
s2 + 1
se−s/2 + πe−s
(ii)
s2 + π 2
3 4e−s 4e−3s
(iii) − 2 + 2
s s s
1
Solution. (i) We have L −1 2
= sint. Since L −1 [e−as F(s)] = f (t − a)u(t − a)
s +1
where f (t) = L −1 [F(s)],
−1 1
L e−πs 2
= sin(t − π) · u(t − π)
s +1
= − sint · u(t − π)
1
s− 1
se 2 + πe−s − s s π
(ii) 2 2
=e 2 2 2
+ e−s 2 . (1)
s +π s +π s + π2
s π
Now L −1 = cos πt and L −1 = sin πt.
s2 + π 2 s2 + π 2
By second shifting property,
1
− s
−1 s 1 1
L e 2 2 = cos π t − ·u t −
s + π2 2 2
1
= sin πt · u t −
2
π
and L −1 e−s · 2 = sin π(t − 1)u(t − 1)
s + π2
= − sin πt · u(t − 1).
Jeeja A. V. [email protected]
31
1
s
−
se 2 + πe−s
By (1), L −1
s2 + π 2
1
= sin πt · u t − − sin πt · u(t − 1)
2
1
= sin πt u t − − u(t − 1)
2
3 1 1
(iii) Let F(s) = − 4e−s · 2 + 4e−3s 2 . (1)
s s s
1 1
Now L −1 = 1, L −1 = t.
s s2
By second shifting properties,
−1 −s 1
L e 2 = (t − 1)u(t − 1) and
s
−1 −3s 1
L e = (t − 3)u(t − 3).
s2
By (1), L −1 [F(s)] = 3 − 4(t − 1)u(t − 1) + 4(t − 3)u(t − 3).
The unit impulse function is considered as the limiting form of the function of
1/k if a ≤ t ≤ a + k
δk (t − a) = (5)
0 otherwise.
This function is represented in the figure. In mechanics, the impuse of a force f (t) over a
time interval, a ≤ t ≤ a + k is defined to be the integral of f (t) from a to a + k. Thus the impulse
Ik of the function (1) is
Z a+k
Ik = δk (t − a)dt
a
Z a+k
1
= dt
a k
1
= (t)a+k
k a
1
= × k = 1.
k
Jeeja A. V. [email protected]
32
When a = 0, L [δ (t)] = e0 = 1.
6 PERIODIC FUNCTIONS
Proof. By definition,
Z ∞
L [ f (t)] = e−st f (t)dt
0
Z T Z 2T Z 3T
−st −st
= e f (t)dt + e f (t)dt + e−st f (t)dt + · · · .
0 T 2T
Jeeja A. V. [email protected]
33
Problem 6.1
Find the Laplace transform of the saw-toothed wave function of period T , defined by
kt
f (t) = , 0 < t < T .
T
Solution. We have
1
Z T
L [ f (t) = e−st f (t)dt
1 − e−sT 0
Z T
1 kt
= −sT
e−st dt
1−e 0 T
Z T
k
= −sT
te−st dt
T (1 − e ) 0
" T Z T −st #
k e−st e
= −sT
t − 1 dt
T (1 − e ) −s 0 0 −s
" −st T #
k Te −sT 1 e
= −sT
+
T (1 − e ) −s s −s 0
−sT
k Te 1 −sT
= − 2 (e − 1) .
T (1 − e−sT ) −s s
Problem 6.2
Find the Laplace transform of the triangular wave function of period 2a given by
t, 0 < t < a
f (t) =
2a − t, a < t < 2a
Solution.
1
Z T
L ( f (t)] = e−st f (t)dt
1 − e−sT 0
Z 2a
1
= e−st f (t)dt
1 − e−s·2a 0
Z a Z 2a
1 −st −st
= e tdt + e (2a − t)dt
1 − e−2as 0 a
( −st a −st 2a )
1 e−st e e−st e
= −2as
t· −1 2
− (2a − t) − (−1)
1−e −s s 0 −s s2 a
1 as
= tanh .
s2 2
Jeeja A. V. [email protected]
34
Problem 6.3
Find the Laplace transform of the Half-wave rectifier function
sin wt f or 0 < t < π/w
f (t) = 2π
0 f or π/w < t <
w
2π
Solution. f (t) is a periodic function with period T = .
w
1
ZT
Let L [ f (t)] = e−st f (t)dt
1 − e−sT 0
Z π/w Z 2π/w
1 −st −st
= e sin wtdt + e odt
2π 0 π/w
−s( )
1−e w
−st π/w
1 e
= (−s sin wt − w cos wt)
2π s2 + w2 0
−s( )
1−e w ( )
1 e−sπ/w · ω w
= + 2 .
−2πs s2 + w2 s + w2
1−e w
7 CONVOLUTION
Also f ∗ g = g ∗ f .
Jeeja A. V. [email protected]
35
Problem 7.1
Apply convolution theorem to evaluate the inverse Laplace transform of the following:
s
(i)
(s2 + a2 )2
s2
(ii)
(s2 + a2 )(s2 + b2 )
1
(iii)
(s2 + a2 )2
s2
(iv)
(s2 + 4)2
1
(v)
s(s2 − a2 )
s s 1 s
Solution. (1) Let = × = f1 (s) f2 (s) where f1 (s) = and
(s2 + a2 )2 s2 + a2 s2 + a2 s2 + a2
1
f2 (s) = .
s2 + a2
Now
f (t) = L −1 [ f1 (s)]
−1 s
=L = cos at
s2 + a2
and
g(t) = L −1 [ f2 (s)]
1
= L −1 [ ]
s2 + a2
sin at
= .
a
By convolution theorem,
−1 s
L 2 2 2
= L −1 [ f1 (s) f2 (s)]
(s + a )
Z t
= f (t − u)g(u)du
0
Z t
sin au
= cos a(t − u) du
0 a
1 t
Z
= [sin at + sin(2au − at)]du
2a 0
t
1 1
= u sin at − cos(2au − at)
2a 2a 0
1
= t sin at.
2a
Jeeja A. V. [email protected]
36
s s
(2) Let f1 (s) = and f 2 (s) =
s2 + a2 s2 + b2
−1 s −1
f (t) = L ( f1 (s)) = L = cos at
s2 + a2
−1 −1 s
g(t) = L ( f2 (s)) = L = cos bt
s2 + b2
s2
−1
∴L
(s2 + a2 )(s2 + b2 )
= L −1 [ f1 (s) f2 (s)]
Z t
= f (t − u)g(u)du, by convolution theorem
0
Z t
= cos a(t − u) cos budu
0
1 t
Z
= [cos(at − (a − b)u) + cos(at − (a + b)u)]du
2 0
1 sin(at − (a − b)u) sin(at − (a + b)u) t
= +
2 −(a − b) −(a + b) 0
1 sin bt − sin at (sin at + sin bt)
=− −
2 a−b a+b
a sin at − b sin bt
=
a2 − b2
1 1
(3) Let f1 (s) = and f 2 (s) = .
s2 + a2 s2 + a2
Then
f (t) = L −1 [ f1 (s)]
−1 1
=L
s2 + a2
sin at
=
a
g(t) = L −1 [ f2 (s)]
−1 1
=L
(s2 + a2 )
sin at
= .
a
Jeeja A. V. [email protected]
37
s2
−1
L = L −1 [ f1 (s) · f2 (s)]
(s2 + 4)2
Z t
= f (t − u) · g(u)du
0
Z t
= cos(2t − 2u) · cos 2udu
0
1 t
Z
= [cos 2t + cos(2t − 4u)]du
2 0
sin(2t − 4u) t
1
= u cos 2t +
2 −4 u=0
1 sin 2t
= t cos 2t +
2 2
1 1
(5) Let f1 (s) = and f2 (s) = 2 .
s s − a2
1
Then f (t) = L [ f1 (s)] = L
−1 −1 = 1 and
s
1 sinh at
g(t) = L −1 [ f2 (s)] = L −1 2 = .
s − a2 a
Jeeja A. V. [email protected]
38
EXERCISE
1 1 3
(i) (ii) (iii)
(s + a)(s + b) s(s2 + 4) (s2 + 1)(s2 + 9)
s s2 2
(iv) (v) (vi)
(s2 + 1)(s2 + 4) s4 − a4 (s + 1)(s2 + 4)
1 1 1
(vii) (viii) (ix)
(s + 1)(s + 9)2 (s − 2)(s − 3) (s + 1)(s2 + 1)
1
(x)
s (s + a2 )
2 2
se−as e−2s
(i) (ii)
s2 − w2 s−3
e−s se−2s e−s
(iii) (iv) (v)
(s − 1)(s − 2) s2 − 1 (s + 1)3
Find the Lapalce transform of the square-wave function of period a defined as f (t) =
(iv)
1 for 0 < t < a/2
.
−1 for a/2 < t < a
2t
(v) Find the Laplace transform of the wave form f (t) = , 0 ≤ t ≤ 3.
3
Jeeja A. V. [email protected]
39
(vi) Find the Laplace transform of the periodic function f (t) = et for 0 < t < 2π.
ANSWERS
e−bt − e−at 1
1. (i) (ii) (1 − cos 2t)
a−b 4
1 1
(iii) [3 sint − sin 3t] (iv) (cost − cos 2t)
8 3
1 2e−t 1
(v) (sinh at + sin at) (vi) − (2 cos 2t − sin 2t)
2a 5 5
e−t
(vii) [1 − e−8t (1 + 8t)] (viii) e2t − e3t
64
1 −t 1
(ix) [e + sint − cost] (x) (at − sin at)
2 a3
e−2s 2 e−4s
2. (i) (4s + 4s + 2) (ii) (cos 4 + s sin 4)
s3 s2 + 1
3. (i) cosh w(t − a) · u(t − a) (ii) e3(t−2) u(t − 2)
1 −(t−1)
(v) e (t − 1)2 u(t − 1)
2
1 as
4. tanh
s 4
2e−3s 2
5. +
−s(1 − e−3s ) 3s2
e2(1−s)π − 1
6.
(1 − s)(1 − e−2sπ )
s
7.
(s2 + w2 )(1 − e−πs/w )
Jeeja A. V. [email protected]
40
Proof. Given
Z ∞
F(s) = L [ f (t) = e−st · f (t) dt (1)
0
= −L [t f (t)].
d
or L [t f (t)] = (−1)1 [F(s)] (2)
ds
d2
Similarly, L [t 2 f (t)] = (−1)2 2 [F(s)]
ds
d3
L [t 3 f (t)] = (−1)3 3 [F(s)].
ds
..
.
dn
In general, L [t n f (t)] = (−1)n [F(s)].
dsn
Problem 8.1
Find the Laplace transform of the following
(ii) t 2 sin wt
(iii) t sinh at
(iv) t 2 et · sin 4t
(vi) t sin2 3t
Jeeja A. V. [email protected]
41
4
(iv) we have L [sin 4t] =
s2 + 16
d2
4
∴ L [t sin 4t] = (−1) 2 2
2 2
ds s + 16
d d 1
= 4×
ds ds s2 + 16
d −2s
= 4×
ds (s2 + 16)2
2
(s + 16)2 1 − s × 2(s2 + 16) × 2s
= −8
(s2 + 16)4
24s2 − 128
= 2
(s + 16)3
Jeeja A. V. [email protected]
42
s
(v) we have L [cos 5t] =
s2 + 25
d s
so L [t cos 5t] = (−1)
ds s2 + 25
2
s2 − 25
(s + 25) − s × 2s
= (−1) =
(s2 + 25)2 (s2 + 25)2
1 − cos 6t
(vi) we have sin2 3t =
2
So
1 1 1 s
L [sin 3t] = [L (1) − L (cos 6t)] =
2
−
2 2 s s2 + 36
d 1 1 s
∴ L [t sin 3t] = (−1)
2
−
ds 2 s s2 + 36
2
1 1 (s + 36)1 − s × 2s
=− − 2−
2 s (s2 + 36)2
36 − s2
1 1
= 2+ ×
2s 2 (s2 + 36)2
Jeeja A. V. [email protected]
43
EXAMPLE 8
Solution
1 1
(i) We have L [1 − et ] = L (1) − L (et ) = −
s s−1
1 − et
Z∞
∴ L = L (1 − et ) ds
t s
Z ∞
1 1
= − ds = {log s − log(s − 1)}∞
s
s s s−1
∞ ∞
s 1
= log = log
s−1 s
1
1− s
s
1
= log 1 − log
1
1−
s
s s−1
= 0 − log = log
s−1 s
Jeeja A. V. [email protected]
44
1 1
(ii) we have L [e−at − e−bt ] = −
s+a s+b
−at
e − e−bt
Z∞
∴ L = L [e−at − e−bt ] ds
t s
Z ∞
1 1
= − ds
s s+a s+b
= {log(s + a) − log(s + b)}∞
s
∞
s+a
= log
s+b s
1 + a/s ∞
= log
1 + b/s s
1 + a/s
= log 1 − log
1 + b/s
s+a
= 0 − log
s+b
s+b
= log
s+a
1
(iii) we have L [sint] =
s2 + 1
Z∞
sint
∴ L = L (sint) ds
t s
1
Z ∞
= 2
ds
s s +1
π
= tan−1 (s)|∞
s = − tan−1 s
2
π
= − tan−1 s
2
= cot−1 (s) = tan−1 (1/s)
Jeeja A. V. [email protected]
45
s s
(iv) we have L [cos at − cos bt] = −
s2 + a2 s2 + b2
Z∞
cos at − cos bt
∴ L = L [cos at − cos bt] ds
t s
Z ∞
s s
= − ds
s s2 + a2 s2 + b2
∞
1 2 2 1 2 2
= log(s + a ) − log(s + b )
2 2 s
2 2 ∞
1 s +a
= log 2
2 s + b2 s
1 1 + a2 /s2 ∞
= log
2 1 + b2 /s2 s
1 + a2 /s2
1
= log 1 − log
2 1 + b2 /s2
2
s + a2
1
= 0 − log 2
2 s + b2
1 s2 + b2
= log 2
2 s + a2
1 s
(v) we have L [1 − cost] = − 2
s s +1
Z∞
1 − cost
∴ L = L[1 − cost] ds
t s
Z ∞
1 s
= − ds
s s s2 + 1
∞
1 2
= log s − log(s + 1)
2 s
1
= [log s2 − log(s2 + 1)]∞ s
2
s2 ∞
1
= log 2
2 s +1 s
∞
1 1 1 1
= log = 0 − log
2 1 + 1/s2 s 2 1 + 1/s2
2 2
1 s 1 s
= − log 2 = − log 2
2 s +1 2 s +1
Jeeja A. V. [email protected]
46
Again
1 − cost
1 − cost t
L =L
t2 t
Z ∞
1 − cost
= L ds
s t
2
1 s
Z ∞
= − log 2 ds
s 2 s +1
2
1 ∞ s
Z
=− log 2 1 ds
2 s s +1
Integrating by parts
2 ∞ Z ∞
1 s 1
=− log 2 s −
2 s +1 s s s2
s2 + 1
[(s2 + 1)2s − s2 · 2s]
× s ds
(s2 + 1)2
∞ Z ∞ 2
2s2
1 1 s +1
=− s log − × 2 ds
2 1 s s s2 (s + 1)2
1+ 2
s Z ∞
1 1 2
=− 0 − s · log − ds
2 1 2
s s +1
1+ 2
2 s
1 s −1 ∞
= − −s log 2 − 2(tan (s))s
2 s +1
2
s s π
= log 2 + − tan−1 s
2 s +1 2
2
s s π
= log 2 + − tan−1 s
2 s +1 2
2
s s
= log 2 + cot−1 s
2 s +1
EXERCISE
sin2 t sinht
(11) (12)
t t
e−t sint
(13) t e−2t cost
2 (14)
t
eat − cos bt
(15)
t
cos 2t − cos 3t
(16) (17) t 2 e−3t sin 2t
t
ANSWERS
If L [ f (t)] = F(s), then L −1 [F(s)] = f (t), where L− 1 is called the inverse Laplace trans-
form operator. From the application point of view, the inverse Laplace transform is very useful.
Important formulae
1
(1) L −1 =1
s
t n−1
1
(2) L −1 =
sn (n − 1)!
1
(3) L −1 = eat
s−a
Jeeja A. V. [email protected]
48
1
(4) L −1 = e−at
s+a
1 sin at
(5) L −1 =
s2 + a2 a
s
(6) L −1 = cos at
s2 + a2
1 sinhat
(7) L −1
2 2
=
s −a a
s
(8) L −1 = cosh at
s2 − a2
(9) Shifting property:
L −1 [F(s − a)] = eat L −1 [F(s)]
(10) Multiplicationby t
d
t f (t) = L −1 − L ( f (t))
ds
(11) Division by t
f (t)
Z ∞
−1
=L L( f (t))ds
t s
EXAMPLE 9
s2 + 2s + 5 2s2 − 6s + 5
(i) (ii)
s3 s3 − 6s2 + 11s − 6
4s + 5 2s + 5
(iii) (iv)
(s + 1)2 (s − 2) 2
s + 4s − 5
2s + 5 5s + 3
(v) 2
(vi)
s + 4s + 13 (s − 1)(s2 + 2s + 5)
s2 + 6 s
(vii) (viii)
(s2 + 1)(s2 + 4) s + 4a4
4
s s+1
(ix) (x) log
s + s2 + 1
4 s−1
1+s −1 s
(xi) log (xii) cot
s 2
Jeeja A. V. [email protected]
49
Solution
s2 + 2s + 5 s2 2s 5 1 2 5
(i) = + + = + +
s3 s3 s3 s3 s s2 s3
2
−1 s + 2s + 5 −1 1 −1 1 −1 1
∴L = L + 2L + 5L
s3 s s2 s3
t t2
= 1+2× +5×
1! 2!
t n−1
−1 n
∵ L (t ) =
(n − 1)!
5 2
= 1 + 2t + t
2
2s2 − 6s + 5 2s2 − 6s + 5
(ii) Let 3 =
s − 6s2 + 11s − 6 (s − 1)(s − 2)(s − 3)
A B C
= + +
s−1 s−2 s−3
Then (by resolving into partial fraction)
2 · 12 − 6 · 1 + 5
Putting s = 1, A = = 1/2
(1 − 2)(1 − 3)
2 · 22 − 6 · 2 + 5
Putting s = 2, B = = −1
(2 − 1)(2 − 3)
2 · 32 − 6 · 3 + 5
Putting s = 3, c = = 5/2.
(3 − 1)(3 − 2)
2s2 − 6s + 5 1/2 −1 5/2
∴ 3 = + +
s − 6s2 + 11s − 6 s−1 s−2 s−3
2s2 − 6s + 5
−1 1 −1 1 −1 1
∴L = L −L
s3 − 6s2 + 11s − 6 2 s−1 s−2
5 1
+ L −1
2 s−3
1 t 5
= e − e2t + e3t
2 2
4s + 5 A B C
(iii) Let = + +
(s + 1)2 (s − 2) s + 1 (s + 1)2 s − 2
Multiplying both sides by (s + 1)2 (s − 2),
Put s = 2, 13 = 9C ⇒ C = 13/9
1
Put s = −1, 1 = −3B ⇒ B = −
3
0 = A +C ⇒ A = −C = −13/9.
Jeeja A. V. [email protected]
50
Put s = 1, 8 = A×8 ⇒ A = 1
Put s = 0, 3 = 5A −C ⇒ C = 5A − 3 = 5 − 3 = 2
Jeeja A. V. [email protected]
51
0 = A + B ⇒ B = −A = −1
5s + 3 1 −s + 2
∴ 2
= + 2
(s − 1)(s + 2s + 5) s − 1 s + 2s + 5
−1 5s + 3 −1 1 −1 −s + 2
L = L +L
(s − 1)(s2 + 2s + 5) s−1 s2 + 2s + 5
−1 1 −1 −(s + 1) + 3
= L +L
s−1 (s + 1)2 − 1 + 5
−1 1 −1 −(s + 1)
= L +L
s−1 (s + 1)2 + 22
−1 1
+3L
(s + 1)2 + 22
−t −1 s −t −1 1
= e −e L
+t
+ 3e L
s2 + 22 s2 + 22
(∵ L −1 [F(s + a)] = e−t L −1 [F(s)])
sin 2t
= et − e−t cos 2t + 3e−t ·
2
s2 + 6
(vii) Let F(s) = .
(s2 + 1)(s2 + 4)
Since F(s) involves only even powers of s, we put s2 = u
u+6 A B
F(s) = = +
(u + 1)(u + 4) u + 1 u + 4
where
−1 + 6 5 −4 + 6 2
A = = and B= =
−1 + 4 3 −4 + 1 −3
5/3 −2/3 5 1 2 1
∴ F(s) = + = · 2 − · 2
u + 1 u + 4 3 s + 1 3 s + 4
5 −1 1 2 1
∴ L −1 [F(s)] = L 2
− L −1 2
3 s +1 3 s +4
5 sint 2 sin 2t
= · − ·
3 1 3 2
5 1
= sint − sin 2t
3 3
(∵ A2 + B2 = (A + B)2 − 2AB)
Jeeja A. V. [email protected]
52
Let
s s
=
s4 + 4a4 (s2 + 2as + 2a2 )(s2 − 2as + 2a2 )
As + B Cs + D
= 2 2
+ 2
s + 2as + 2a s − 2as + 2a2
−1/4a 1/4a
= 2 2
+ 2
s + 2as + 2a s − 2as + 2a2
1 1 1
=− × 2 2 2
+
4a (s + a) − a + 2a 4a
1
×
(s − a) − a2 + 2a2
2
1 1 1 1
=− × 2 2
+ ×
4a (s + a) + a 4a (s − a)2 + a2
−1 s 1 −at −1 1 1
L 4 4
= − ×e ×L 2 2
+
s + 4a 4a s +a 4a
1
× eat L −1 2
s + a2
1 sin at 1 sin at
= − e−at + eat
4a a 4a a
sin at at sin at
= (e − e−at ) = × sinh at
4a2 2a2
= (s2 + 1)2 − s2
= (s2 + 1 + s)(s2 + 1 − s)
Let
s s As + B Cs + D
= = +
s4 + s2 + 1 (s2 + s + 1)(s2 − s + 1) s2 + s + 1 s2 − s + 1
Multiplying both sides by s4 + s2 + 1,
O = B − A +C + D (2)
Jeeja A. V. [email protected]
53
and hence
C = O.
From (3),
1 = D−B
and by (4),
O = D+B
Adding
1 1
1 = 2D ⇒ D = and B = −D = −
2 2
s −1/2 1/2
∴ = +
s4 + s2 + 1 s2 + s + 1 s2 − s + 1
−1/2 1/2
= +
1 1 1 1
(s + )2 − + 1 (s − )2 − + 1
2 4 2 4
−1/2 1/2
= +
1 2 3 1 3
(s + ) + (s − )2 +
2 4 " 2 4 #
s 1 1
∴ L −1 4 2 = − × e−t/2 L −1 √ 2
s +s +1 2 s2 + 3/2
1 t/2 −1 1
+ e L √
2 s2 + ( 3/2)2
√ √
1 −t/2 sin 3t/2 1 t/2 sin 3t/2
=− e √ + ×e × √
2 3/2 2 3/2
√ !
2 3 et/2 − e−t/2
= √ sin t
3 2 2
√
2 3 t
= √ sin t sinh
3 2 2
Jeeja A. V. [email protected]
54
s+1
(x) Let L [ f (t)] = F(s) = log = log(s + 1) − log(s − 1)
s−1
Then
d
L [t f (t)] = − L [ f (t)]
ds
d
= − [log(s + 1) − log(s − 1)]
ds
1 1
= − −
s+1 s−1
1 1
= −
s−1 s+1
−1 1 1
∴ t f (t) = L − = et − e−t
s−1 s+1
t
e −e −t
s+1
et − e−t
−1
∴ f (t) = or L log =
t s−1 t
s+1
(xi) Let L [ f (t)] = F(s) = log = log(s + 1) − log s
s
Then
−1 s+1
f (t) = L log
s
d
But L [t f (t)] = −
L [ f (t)]
ds
d
= − [log(s + 1) − log(s)
ds
1 1
=− −
s+1 s
1 1
= −
s s+ 1
−1 1 1
∴ t f (t) =L − = 1 − e−t
s s+1
1 − e−t
∴ f (t) =
t
e−t
s + 1 1 −
or L −1 log =
s t
Then
f (t) = L −1 [cot−1 (s/2)]
Jeeja A. V. [email protected]
55
But
d
L [t f (t)] = − L [ f (t)]
ds
d
= − [cot−1 (s/2)]
ds
−2 2
= − 2 2
= 2
s +2 s + 22
2
∴ t f (t) = L −1 2 = sin 2t
s + 22
sin 2t
∴ f (t) =
t
EXERCISE 4.3
ANSWERS
Jeeja A. V. [email protected]
56
1 √ √ !
3 3 2 t4 1 − t √ 3 3
(i) − t + (ii) √ e 2 2 cos t + sin t
2 2 16 3 2 2
7 3t 1 −t 4 2t 3
(iii) e − e − e (iv) sin 2t − sint
2 6 3 2
3 cosh at + cos at
(v) 2e−t − 2e2t + 6te2t + t 2 e2t (vi)
2 2
t(e − e−2t )
t
(vii) e−2t (cost + 6 sint) (viii)
3
2(1 − cost) 1
(ix) (x) e2t (3 cos 4t − sin 4t)
t 2
1 −t 1 3t 1
(xi) te sint (xii) e cos 2t − e3t sin 2t
2 4 8
et sint
(xiii) sin 4t (xiv)
8 t
2 −t
e sint
(xv) (1 − cosht) (xvi)
t t
e−at − cos bt
(xvii) (xviii) e−t (1 − cost)
t
1
(xix) t sinht (xx) 3e−3t t sint
2
1 3
(xxi) (1 + e−t ) sint + (1 − e−t ) cost
5 5
e−bt − e−at 3
(xxii) (xxiii) 2e−t + 5e2t − et/2
t 2
REFERENCES
[1] Erwin Kreyszig, Advanced Engineering Mathematics, 10th Edition, Wiley-India
Jeeja A. V. [email protected]